SlideShare a Scribd company logo
Tuberculosi
s
Done by :-Mohammed Al-Haidary
Historical background
 TB is an ancient disease.
 Signs of skeletal TB (Pott disease) have been found
in remains from Europe from Neolithic times (8000
BCE), ancient Egypt (1000 BCE),
 German physician Robert Koch discovered and
isolated M tuberculosis in 1882.
 by the Industrial Revolution in Europe (1750), it was
responsible for more than 25% of adult deaths.
1882
Epidemiology
 TB SITUATION AND RESPONSE
 Tuberculosis (TB) is contagious and airborne.
 TB is the second leading infectious killer after
COVID-19 and the 13th leading cause of death
worldwide. It is also the leading killer of people with
HIV and a major cause of deaths related to
antimicrobial resistance
 One-fourth of the world's population has latent TB.
Etiology
 Mycobacterium species that cause tuberculosis are collectively known as the
Mycobacterium tuberculosis complex, which includes:
 Mycobacterium tuberculosis
 Mode of transmission: spread via aerosol droplet nuclei
 Reservoir: predominantly humans
 Disease: all forms of tuberculosis
 Mycobacterium bovis
 Mode of transmission: predominantly via ingestion of contaminated cow's milk
 Reservoir: predominantly cattle
 Disease: gastrointestinal tuberculosis in humans
 Mycobacterium africanum.
 Mycobacterium microti.
Etiology
 Mycobacteria other than tuberculosis (MOTT) most often
cause disease in individuals with weakened immune systems
 Mycobacterium avium and M. intracellulare are the more
common MOTT sometimes seen in patients co-infected with
HIV
Etiology
 Features of Mycobacterium tuberculosis
 Type: facultative intracellular rod-shaped bacteria
 Gram stain: does not stain well
 Special stains
1. Ziehl-Neelsen stain: acid-fast bacilli appear pink
2. Auramine-rhodamine stain
• Acid-fast bacilli appear reddish-yellow on fluorescence microscopy.
• Used as a screening tool because of high sensitivity and low cost
 Culture mediums for growth
1. Löwenstein Jensen medium
2. Middlebrook medium
3. Rapid automated broth culture
Types of tuberculosis
 Drug-resistant tuberculosis
 Definition: a form of TB that is resistant to one or more antitubercular agents
 Types
1. Rifampin-resistant tuberculosis (RR-TB): resistance to rifampin with or without resistance
to other antitubercular drugs
2. Monoresistant TB: resistance to one of the first-line antitubercular drugs
3. Multidrug-resistant tuberculosis (MDR-TB): resistance to both isoniazid and rifampin
4. Extensively drug-resistant tuberculosis (XDR-TB): resistance to any single
fluoroquinolone and at least one of the second-line injectable drugs (e.g., amikacin,
kanamycin, or capreomycin) in addition to MDR-TB
 Causes
1. Incorrect drug combination therapy
2. Inadequate duration or dosage of drug therapy
3. Poor treatment adherence
4. Poor quality of drugs
5. Close contact with an individual with drug-resistant TB
FROM EXPOSURE TO INFECTION
 M. tuberculosis is most commonly transmitted from a person with infectious
pulmonary TB by droplet nuclei containing M. tuberculosis bacteria, which are
aerosolized by coughing, sneezing, or speaking. The tiny droplets dry rapidly;
the smallest (<5–10 μm in diameter) may remain suspended in the air for
several hours and may reach the terminal air passages when inhaled. There
may be as many as 3000 infectious nuclei per cough.
 Other routes of transmission of tubercle bacilli (e.g., through the skin or the
placenta) are uncommon and of no epidemiologic significance.
 The risk of transmission and of subsequent acquisition of M. tuberculosis
infection is determined mainly by exogenous factors, although endogenous
factors may also play a role. The probability of contact with a person who has
an infectious form of TB, the intimacy and duration of that contact, the degree
of infectiousness of the case, and the shared environment in which the contact
takes place are all important determinants of the likelihood of transmission.
FROM EXPOSURE TO INFECTION
 Several studies of close-contact situations have clearly
demonstrated that TB patients whose sputum contains AFB
visible by microscopy (sputum smear–positive cases) are the
most likely to transmit the infection. The most infectious
patients have cavitary pulmonary disease or, much less
commonly, laryngeal TB and produce sputum containing as
many as 105–107 AFB/mL.
 Patients with sputum smear–negative/culture-positive TB are
less infectious, although they have been responsible for up to
20% of transmission in some studies in the United States.
 the majority of inhaled
bacilli are trapped in the
upper airways and expelled
by ciliated mucosal cells, a
fraction (usually <10%)
reach the alveoli, a unique
immunoregulatory
environment.
Pathophysiology
1. Primary tuberculosis
 Innate immune response
 Exposure to M. tuberculosis
1. Individuals with M. tuberculosis infection disperse droplet nuclei that contain bacilli via
sneezing or coughing.
2. Inhaled droplet nuclei reach the terminal alveoli and are taken up by the alveolar
macrophages.
 Entry into macrophages
1. Mycobacterial cell wall contains pathogen-associated molecular patterns (PAMPs) such
as lipoarabinomannan and lipomannan.
2. Alveolar macrophages recognize M. tuberculosis PAMPs via toll-like receptors (TLRs)
3. Activation of TLRs leads to the production of proinflammatory cytokines (e.g., IL-1, IL-12,
TNF-α) and phagocytosis of mycobacteria.
Pathophysiology
 Replication within macrophages
 Typically, phagocytosed organisms reside within a phagosome
to undergo intracellular killing via the following steps:
1. Phagosome maturation: acidification using a proton pump
system
2. Fusion of phagosome and lysosome: mediated by increased
intracellular calcium levels
3. Killing of bacteria by reactive oxygen species (ROS), reactive
nitrogen intermediates (RNI), and lysosomal enzymes
 M. tuberculosis survives within macrophages because of the
inhibition of both phagosome maturation and phagolysosome
fusion
Pathophysiology
 Virulence factors involved include:
 Cord factor: This is a surface glycolipid that causes the
bacterium to grow in a serpentine or cord-like pattern. It inhibits
the migration of neutrophils and induces the release of TNF-α,
a pro-inflammatory cytokine. TNF-α stimulates activated
macrophages to form granulomas, which are organized
clusters of immune cells that help contain the infection.
 Sulfatides: These are surface glycolipids that inhibit the fusion
of the phagosome with lysosomes, preventing the formation of
a fully functional phagolysosome.
Pathophysiology
 Lipoarabinomannan: This is a lipoglycan, a complex molecule
composed of lipids and sugars, that is found on the surface of M.
tuberculosis. It induces the release of TNF-α from macrophages,
contributing to the inflammatory response. Additionally,
lipoarabinomannan can scavenge reactive oxygen species (ROS),
which are toxic molecules produced by immune cells to destroy
bacteria.
 Catalase-peroxidase: This enzyme is produced by M. tuberculosis
and helps the bacterium survive within the macrophage by catalyzing
the destruction of ROS and hydrogen peroxide (H2O2). ROS and
H2O2 are produced by immune cells as part of their defense
mechanism against bacterial infections.
Pathophysiology
 Macrophage lysis and release of bacteria
1. Following replication in the alveolar macrophages,
the released bacteria attack uninfected
macrophages to spread infection.
2. Dendritic cells migrate to the site of infection and
process mycobacterial antigens.
3. Some bacteria enter the bloodstream, causing
bacteremia and seeding multiple organs.
Pathophysiology
 Cellular immune response
 Th1 cell activation
1. Dendritic cells present mycobacterial antigens complexed with MHC 2
to naive T cells
2. Activated CD4+ T cells migrate to the focus of infection (type IV
HSR).
 Macrophage activation and bacterial killing
• Activated CD4+ T cells release IFN-γ
• IFN-γ acts on macrophages to enable bacterial killing via the following
mechanisms:
1. Promotion of phagosome maturation
2. Enhanced RNI production
3. Autophagy
Pathophysiology
 Granulomatous inflammation and tissue destruction
1. IFN-γ-activated macrophages secrete TNF-α.
2. TNF-α promotes the aggregation of macrophages and T cells to form
granulomas, affecting the lungs and regional lymph nodes
3. Destruction of M. tuberculosis-infected macrophages causes central
caseous necrosis and tissue damage.
4. Granuloma limits the spread of infection.
5. Ghon focus: a granuloma typically located in the middle/lower lung
lobes.
6. Ghon complex: formed by the Ghon focus, regional lymph node, and
the linking lymphatic vessels
Pathophysiology
 Disease progression
 Sufficient immune response
 Most of the bacteria are killed.
 Some bacteria may persist, causing LTBI.
 The granulomas in the Ghon complex undergo fibrosis and
calcification to form the Ranke complex.
 Deficient immune response (e.g., HIV, malnutrition)
 Failure of granulomas to limit infection
 Progressive primary TB causing progressive lung disease,
bacteremia, and miliary TB
Pathophysiology
2. Secondary tuberculosis
 Latent TB: Dynamic equilibrium is maintained between the host immune response and M.
tuberculosis.
 Reactivation of disease: due to weakening of immune response (e.g., resulting from HIV,
TNF-α inhibitor therapy)
 Disease progression
 Caseating granulomas with central necrosis and Langhans giant cells are characteristic
features.
 Usually affects the upper lobes of the lungs because of higher oxygen tension
 Can also affect other organs (due to seeding of organs in primary tuberculosis)
 Prior sensitization to mycobacterial antigens results in a stronger inflammatory response,
causing extensive tissue destruction, cavitation, and scarring.
Clinical features
 Pulmonary tuberculosis
 Systemic
1. Low-grade fever with night sweats
2. Weight loss (often severe), anorexia
3. Decreased appetite
4. Malaise, weakness
 Pulmonary
1. Non-productive cough
2. Symptoms of progression: productive cough with purulent sputum,
hemoptysis
3. Shortness of breath
4. Pleuritic chest pain
Clinical examination
 Findings are nonspecific.
 General: pallor, clubbing (advanced disease), generalized wasting
 Chest examination: Findings vary depending on the type and degree
of pulmonary involvement.
1. Consolidation: dullness on percussion, crackles, diminished breath
sounds
2. Cavitation: hyperresonance on percussion, amphoric breath sounds
3. Bronchial obstruction: rhonchi
4. Pleural effusion and/or empyema: dullness on percussion, diminished
breath sounds
 Other possible findings: erythema nodosum, phlyctenular
conjunctivitis
DIAGNOSTIC
Diagnostics
 Microbiological studies
 Confirmation of the presence of tuberculosis bacilli in different
samples is done by direct visualization, positive culture, or by
detecting genetic material. Samples used for testing include:
1. Sputum samples (most common): Obtain ≥ 3 samples separated by 8
to 24-hour intervals (should include at least one early morning
sample).
2. Induced sputum
3. Gastric lavage
4. Bronchoalveolar lavage
5. Extrapulmonary TB suspected: Fluid specimens or tissue samples
should be sent for cell count, chemistries, AFB smear microscopy,
and NAAT
Diagnostics
 Drug susceptibility testing
 Standard culture: used to assess for drug susceptibility in most
patients; results are available after several weeks.
 Rapid molecular testing : for select patients at high risk of resistant
tuberculosis
1. History of previously treated tuberculosis
2. Contact with individuals with MDR-TB
3. HIV infection
4. Patients born or having lived for ≥ 1 year in a country with moderate
to high TB incidence or high MDR-TB prevalence
HIV and TB coinfection diagnosis
 Diagnosing TB in patients with HIV coinfection is challenging because
these individuals often have a negative AFB smear and commonly
have atypical imaging findings resulting from paucibacillary disease
due to a reduced immune response. Different measures can be used
to increase diagnostic sensitivity.
 Advanced testing in patients with HIV (not widely available)
1. For most patients: fluorescent AFB smear microscopy , specialized
culture mediums
2. For patients with severe disease and those with a CD4 count < 100
cells/mm3: lateral flow urine lipoarabinomannan assay>>>>>A
nonsputum-based diagnostic test used to detect mycobacterial
antigens in urine. Mainly used for HIV patients with severe disease
and/or those with a CD4 count < 100 cells/mm³ for a rapid point-of-
care diagnosis.
TREATMENT
 Chemotherapy for TB became possible with
the discovery of streptomycin in 1943.
Treatment
 General principles
 Goals
1. Reduction of disease severity and risk of transmission
2. Eradication of all bacilli to achieve sustained cure without relapse
following completion of treatment
3. Prevention of drug resistance during therapy
 Infection control measures
1. Case notification: mandatory reporting to local health department
2. Airborne precautions: should include a surgical mask for the patient
and adequate personal protective equipment (including respirators)
for medical staff and caregivers
3. Contact tracing.
Treatment
 Pretreatment evaluation
 Clinical assessment
1. Nutritional assessment
2. Symptom review
3. Eye exam: visual acuity and color discrimination
 Microbiology: sputum smear microscopy and culture, drug
susceptibility testing
 Imaging: chest x-ray or other chest imaging
 Laboratory studies
1. All patients: liver chemistries, platelet count, creatinine, HIV screening
2. Depending on risk: hepatitis B and hepatitis C screening, diabetes
screen
1-Treatment of drug-susceptible TB
 Intensive phase: 2 months of rifampin PLUS isoniazid, pyrazinamide, and
ethambutol
 The intensive phase allows for an increased antibiotic serum concentration,
optimizes the killing of bacteria, and minimizes the risk of developing antibiotic
resistance.
 Continuation phase: 4 months of rifampin PLUS isoniazid
 The continuation phase reduces the risk of relapse. If an individual has both
cavitating disease and a positive sputum smear microscopy or culture at the
end of the intensive phase of treatment, the continuation phase can be
extended by three months (a total of seven months of continuation).
 Adjuvant treatment: Pyridoxine to prevent vitamin B6 deficiency resulting from
isoniazid (promotes pyridoxine excretion) for all individuals at risk of
neuropathy
1-Treatment of drug-susceptible TB
 Standard regimens are divided into an
intensive (bactericidal) phase and a
continuation (sterilizing) phase. During the
intensive phase, the majority of tubercle
bacilli are killed, symptoms resolve, and
usually the patient becomes noninfectious.
The continuation phase is required to
eliminate persisting mycobacteria and
prevent relapse.
Directly observed therapy
 Directly observed therapy (DOT; standard
practice): Health care personnel observe the
patient taking the medication.
 Associated with increased treatment
success, medication adherence, and prompt
recognition of medication side effects
 Restart full treatment if there is an
interruption of ≥ 14 days during the intensive
phase.
2-Treatment of MDR-TB
 Drug-resistant TB may be either primary or
acquired. In primary drug resistance, the patient is
infected from the start by a drugresistant strain.
Acquired resistance develops in the infecting strain
during treatment.
 Because there is no cross-resistance among the commonly used classes of
drugs, the probability that a strain will be resistant to two drug classes is the
product of the probabilities of resistance to each drug class and thus is low.
2-Treatment of MDR-TB
 MDR-TB, in which bacilli are resistant to (at least) isoniazid and
rifampin, is more difficult to manage than is disease caused by drug-
susceptible organisms because these two bactericidal drugs are the
most potent first-line agents available and because associated
resistance to other first-line drugs as well (e.g., ethambutol) is not
uncommon.
 As a result, two main approaches are now recommended by the WHO
to treat MDR/RR-TB:
1. an individualized longer regimen of 18–20 months’ duration
consisting of an optimal combination of oral drugs chosen according
to a rational approach and using the WHO priority grouping of
medicines
2. a shorter, all-oral, bedaquiline-containing regimen of 9–12 months’
duration
2-Treatment of MDR-TB
1. Longer MDR-TB Regimen In MDR/RR-TB patients where
infecting strains have or are presumed to have additional
resistance (e.g., resistance to the fluoroquinolones), in those
who have severe pulmonary or extrapulmonary disease, or
those who have been treated previously with second-line
drugs, a longer regimen is recommended.
2. Shorter MDR-TB Regimen In MDR/RR-TB patients with no
extensive pulmonary disease or severe extrapulmonary
disease, without history of previous treatment with second-line
drugs, and whose infecting strains are not resistant to the
fluoroquinolones, a shorter, all-oral, bedaquiline-containing
regimen is recommended.
Monitoring
 Monthly follow-ups are recommended in all patients receiving antituberculosis treatment
for active TB or LTBI.
 All patients
1. Assess adherence to therapy.
2. Perform a symptom review and screen for clinical features of active TB.
3. Monitor for side effects.
 Patients with active TB
1. Sputum smear microscopy and culture( Obtain monthly until two consecutive specimens
are negative)
2. Weight and vision assessment(Vision assessment needs to be performed monthly for at
least the first three months.)
3. Laboratory studies: If indicated, obtain liver chemistries, platelet count, and creatinine
Monitoring
 With the recommended 6-month standard first-line regimen, >80% of drug-susceptible TB
patients will have negative sputum cultures at the end of the second month of treatment.
By the end of the third month, the sputum of virtually all patients should be culture
negative.
 patients with cavitary disease in whom sputum culture conversion does not occur by 2
months require immediate testing or retesting for drug resistance. When a patient’s
sputum cultures or smears remain positive at ≥3 months despite good adherence,
treatment failure caused by drug resistance is likely.
 A sputum specimen should be collected at the end of treatment to document cure
 Bacteriologic monitoring of patients with extrapulmonary TB is more difficult and often is
not feasible. In these cases, the response to treatment must be assessed clinically with
the help of medical imaging.
Definitions
 New: a patient who has never had treatment for TB or who has take
anti-tuberculosis drugs for less than 1 month.
 Relapse: a patient previously treated for TB, who has been declared
cured or treatment completed, and is diagnosed with bacteriologically
positive (smear or culture) TB.
 Treatment after failure: a patient who is started on a re-treatment
regimen after having failed previous treatment.
 Treatment after default: a patient who returns to treatment, positive bacteriologically,
following interruption of treatment for 2 months or more.
 Transfer in: a patient who has been transferred from another TB register to continue
treatment.
 Others: all cases that do not fit the above definitions. This group includes chronic case, a
patient who is sputum-positive at the end of a re-treatment regimen.
NOTE
Most initially infectious
patients become noninfectious
within 2 weeks of starting
effective treatment, and many
become noninfectious within
several days.
Latent tuberculosis
infection
Description
 A state of constant immune response
stimulation due to M. tuberculosis
antigens, with no signs of active TB
infection
 Individuals with latent TB are
asymptomatic and not contagious.
Diagnosis of LTBI
 The decision of whether to test an individual
should be carefully considered based upon:-
1. the likelihood of someone having LTBI.
2. the likelihood of progression of LTBI to
active TB.
3. the potential benefit of therapy.
Diagnosis of LTBI
 Individuals with a high likelihood of LTBI
1. Known contact with AFB smear-positive individual
2. Individuals born in or former residents of countries with a high
TB burden (> 20 cases per 100,000 population)
3. People who live or work in high-risk settings (e.g., correctional
facilities, long-term care facilities or nursing homes, homeless
shelters)
4. Health-care workers who care for patients at increased risk for
TB disease
5. Children (< 18 years of age) exposed to adults at increased
risk for tuberculosis infection
Diagnosis of LTBI
 Individuals with a high likelihood of progression from LTBI to
active TB
 HIV infection
 Immunosuppressive therapy, including:
 Corticosteroid therapy
 TNF-α inhibitor therapy (e.g., infliximab)
 Others: methotrexate, cyclosporine; drugs used for organ
transplant and stem cell transplant recipients
 Silicosis
Diagnosis of LTBI
 Age < 5 years
 Prior TB infection
 Smoking
 Heavy alcohol consumption
 Malnutrition, low body mass index
 Advanced age
 Malignancy
 Gastrectomy or jejunoileal bypass
Interpretation of results
 IGRA
1. Positive: TB infection is likely
2. Negative: TB infection is unlikely, but cannot be excluded
3. Indeterminate: can occur in immunosuppressed states, and a repeat IGRA or TST can be
useful
 TST: Depending on patient characteristics, a TST can be positive with an induration ≥ 5
mm, ≥ 10 mm, or ≥ 15 mm.
 For healthy individuals with no risk factors, an induration < 15 mm is considered negative
for TB.
Treatment
1. Pharmacological therapy: indicated for
patients with positive IGRA or TST after
active TB has been excluded
2. Select a regimen based on the length of
treatment, drug tolerance, pharmacological
interactions, and HIV status.
The primary goal of the
treatment of latent TB is
to prevent reactivation to
active TB.
NOTE
Extrapulmonary tuberculosis
Tuberculous lymphadenitis
 Pathophysiology
1. Hematogenous dissemination following primary TB
2. Local extension of infection from the affected organ
 Sites of lymph node involvement
1. Cervical
2. Inguinal
3. Axillary
Tuberculous lymphadenitis
 Clinical features
 Constitutional symptoms
 Lymphadenopathy: Findings vary depending on the stage of lymphadenitis.
1. Lymphadenitis: firm, mobile, and discrete lymph nodes
2. Periadenitis: rubbery and fixed lymph nodes
3. Cold abscess: soft and fluctuant lymph nodes
4. Draining sinus tract
 Diagnostics
1. Ultrasound or CT: matted lymph nodes with a necrotic center
2. FNAC: acid-fast staining, PCR, and culture
3. Lymph node biopsy: histopathology
 Treatment: See “Treatment of active TB.”
Tuberculous pleurisy
 Pathophysiology
1. Direct spread of infection from the lungs
2. Hematogenous spread
3. Delayed hypersensitivity reaction in the pleural space due to mycobacterial antigen
 Clinical features
1. Constitutional symptoms
2. Nonproductive cough
3. Intensive (pleuritic) chest pain
4. Dyspnea
Tuberculous pleurisy
 Diagnostics
 Imaging
1. Chest x-ray: unilateral pleural effusion
2. Chest ultrasound or CT: loculated effusion with septations; pleural thickening
 Thoracentesis: pleural fluid analysis
1. Exudative type (Light criteria)
2. pH: < 7.4
3. Glucose: < 60 mg/dL
4. ↑ Adenosine deaminase
5. Rich in lymphocytes
6. Microbiology: acid-fast staining, PCR, and culture
 Sputum smear examination: acid-fast staining
1. Treatment
 Therapeutic thoracentesis for symptom relief
Pericardial tuberculosis
 Pathophysiology
1. Hematogenous spread from the site of primary infection
2. Retrograde lymphatic spread from hilar lymph nodes
 Stages
1. Stage 1: fibrinous exudate containing neutrophils and
mycobacteria
2. Stage 2: serosanguineous effusion with lymphocytes
3. Stage 3: absorption of effusion and caseating granulomas
4. Stage 4: scarring with calcification
Pericardial tuberculosis
 Types
1. Pericardial effusion (rarely, cardiac tamponade)
2. Constrictive pericarditis
3. Effusive-constrictive pericarditis
 Clinical features
1. Constitutional symptoms
2. Chest pain
3. Cough
4. Dyspnea
Pericardial tuberculosis
 Physical examination findings: See “Clinical features” in “Pericardial effusion and cardiac
tamponade” and “Pericarditis.”
 Diagnostics
 ECG: nonspecific ST-T changes
 Imaging
1. Chest x-ray: enlarged cardiac silhouette or calcification
2. Echocardiography: effusion or diminished wall motility
 Pericardiocentesis
1. Analysis: exudative, blood-stained
2. Microbiology: acid-fast staining, PCR, and culture
3. ↑ Adenosine deaminase
 Treatment
1. See “Treatment of active TB.”
2. Therapeutic pericardiocentesis
Miliary tuberculosis
 Definition: a form of TB with multiorgan involvement that manifests with granulomatous
lesions (resembling millets) as a result of lymphohematogenous dissemination of bacilli
from a pulmonary or extrapulmonary source
 Epidemiology: ∼ 20% of extrapulmonary TB cases
 Pathophysiology: primary infection or reactivation
 Common sites of involvement: lungs, spleen, liver, lymph nodes, adrenals, meninges,
vertebrae (Pott disease), joints and long bones, and choroid
 Clinical features: mostly nonspecific
1. Cough and dyspnea
2. Constitutional symptoms
3. Lymphadenopathy
4. Hepatosplenomegaly
5. Tuberculosis miliaria cutis disseminata
Miliary tuberculosis
 Diagnostics: requires a high index of clinical suspicion
 Fundoscopy: choroid tubercles
 Microbiology: sputum, body fluids, and tissue
1. Acid-fast staining
2. PCR
3. Culture
 Blood
1. ↑ ESR and CRP
2. CBC: anemia, leukopenia
3. LFT: elevated transaminases and bilirubin
4. Electrolytes: hyponatremia, hypercalcemia
 TST: negative
Miliary tuberculosis
 Imaging
1. Chest x-ray: multiple small nodules (< 2 mm) with an appearance resembling millet seeds
2. Ultrasound: pleural effusion, ascites, hepatic or splenic lesions
3. CT: lung nodules, enlarged hilar lymph nodes
4. Brain MRI: meningeal and/or basal cistern enhancement, hydrocephalus, tuberculoma
5. Echocardiography: pericardial effusion
 Treatment: See “Treatment of active TB.”
Gastrointestinal tuberculosis
 Pathophysiology
1. Ingestion of infected milk or sputum
2. Hematogenous spread resulting from primary
pulmonary TB
3. Contiguous spread via affected lymph nodes
 Sites of involvement: See “Types of gastrointestinal
TB” below.
Treatment
1. Surgical: for intestinal obstruction, perforation,
fistula, abscess
2. Medical: See “Treatment of active TB.”
Renal and urologic TB
 Pathophysiology
1. Hematogenous spread of infection to the kidneys as a result of
primary pulmonary TB that is reactivated in
immunosuppressed states
2. Ureters and bladder are infected when bacteria are excreted in
urine.
 Common sites of involvement
1. Kidneys
2. Ureters
3. Bladder
Renal and urologic TB
 Clinical features
 Constitutional symptoms
 Flank pain
 Hematuria
 Storage symptoms
1. Increased frequency of voiding
2. Nocturia
3. Urgency
 Hypertension
 Recurrent UTIs unresponsive to antibiotic therapy
Renal and urologic TB
 Diagnostics
 Urine examination
1. Analysis: hematuria, sterile pyuria, proteinuria
2. Microbiology: acid-fast staining, PCR, and culture
 Imaging: CT or IV urography
1. Renal scarring
2. Hydroureteronephrosis
3. Calcification involving the entire urinary tract
4. Multiple ureteral strictures
5. Bladder wall thickening and fibrosis
 Cystoscopy
1. Ulceration
2. Granuloma
3. Fibrosis
Renal and urologic TB
 Treatment
 Medical: See “Treatment of active TB.”
 Ureteral stenting for strictures
 Surgery
1. Total or partial nephrectomy
2. Bladder augmentation
Male genital tract TB
 Pathophysiology
1. Hematogenous spread of infection to the epididymis and prostate
2. Extension of infection from the epididymis to the testis, vas deferens, seminal vesicles,
and ejaculatory ducts
 Common sites of involvement
1. Epididymis
2. Testis
3. Prostate
4. Vas deferens, seminal vesicles, and ejaculatory duct
 Clinical features
1. Scrotal mass (can be painful or painless)
2. Scrotal sinus tract with watery discharge
3. Infertility
4. Increased urinary frequency, nocturia, and hematospermia
5. Recurrent prostatitis or epididymitis unresponsive to antibiotic therapy
Male genital tract TB
 Diagnostics
 Urine examination
1. Analysis: hematuria, sterile pyuria
2. Microbiology: acid-fast staining, PCR, and culture
 Scrotal ultrasound
1. Calcification
2. Hydrocele
3. Diffuse or nodular enlargement of the epididymis
 Transrectal ultrasound: calcification in prostate and seminal
vesicles
 Treatment: See "Treatment of active TB.”
Female genital tract TB
 Pathophysiology: hematogenous spread of infection as a result of primary pulmonary TB
that is reactivated in immunosuppressed states
 Common sites of involvement
1. Ovaries
2. Fallopian tubes
3. Endometrium
 Clinical features
1. Constitutional symptoms
2. Menstrual irregularities
3. Abdominal pain
4. Infertility
5. Adnexal mass on pelvic examination
Female genital tract TB
 Diagnostics
 Menstrual fluid or endometrial curettage sample: acid-fast staining,
PCR, and culture
 Imaging
1. Ultrasound/CT: tuboovarian abscess
2. Hysterosalpingography: occluded fallopian tubes, hydrosalpinx,
calcification
3. Hysteroscopy: adhesions and obliterated uterine cavity
 Treatment
1. Medical: See “Treatment of active TB.”
2. Surgery: drainage of tuboovarian abscess
Complications
Complications
1. Rasmussen aneurysm: inflammatory pseudoaneurysm of a branch of the pulmonary
artery lying adjacent to a tuberculous cavity and manifesting with hemoptysis
2. Massive hemoptysis: due to erosion of blood vessels overlying a lung cavity, Rasmussen
aneurysm, or aspergilloma
3. Lung cavitation
4. Lung fibrosis
5. Aspergilloma
6. Pneumothorax
7. Bronchiectasis
8. Fibrosing mediastinitis
9. Venous thromboembolism
Prevention
 BCG was derived from an attenuated strain of M.
bovis and was first administered to humans in
1921
Prevention
 The primary way to prevent TB is to diagnose and isolate
infectious cases rapidly and to administer appropriate
treatment until patients are rendered noninfectious (usually 2–4
weeks after the start of proper treatment) and the disease is
cured. Additional strategies include BCG vaccination and
preventive treatment of persons with TB infection who are at
high risk of developing active disease.
BCG VACCINATION
1. Historically one of the most used vaccines in the history of medicine, BCG was derived
from an attenuated strain of M. bovis and was first administered to humans in 1921
2. Many BCG vaccines are available worldwide; all are derived from the original strain, but
the vaccines vary in efficacy, ranging from 80% to nil.
3. These studies and a meta-analysis also found higher rates of efficacy in the protection of
infants and young children from serious disseminated forms of childhood TB, such as
tuberculous meningitis and miliary.
4. TB. BCG vaccine is safe and rarely causes serious complications. The local tissue
response begins 2–3 weeks after vaccination, with scar formation and healing within 3
months
BCG VACCINATION
 Composition: live attenuated strain of M. bovis
 Administration: 0.3 mL of reconstituted vaccine using a multiple puncture device over the
deltoid region
 Indications
1. Countries with a high TB burden: all newborns at birth
2. Countries with a low TB burden
 Children with a negative TST and who have come into contact with adults with
untreated/inadequately treated TB or drug-resistant TB (if the child cannot take long-term
medication for infection)
 Health care workers continually exposed to individuals with drug-resistant TB
 Contraindications
1. Immunocompromised individuals
2. Pregnancy
 Other uses: intravesical chemotherapy in superficial bladder cancer
BCG VACCINATION
 Side effects
1. most commonly, ulceration at the vaccination site and regional
lymphadenitis—occur in 1–10% of vaccinated persons.
2. Some vaccine strains have caused osteomyelitis in ~1 case per
million doses administered.
3. Disseminated BCG infection (“BCGitis”) and death have occurred in
1–10 cases per 10 million doses administered, although this problem
is restricted almost exclusively to persons with impaired immunity,
such as children with severe combined immunodeficiency syndrome
or adults with HIV infection.
BCG VACCINATION
 BCG vaccine is recommended for routine use at
birth in countries or among populations with high TB
prevalence . However, because of the low risk of
transmission of TB in the United States and other
high-income countries, the variability in protection
afforded by BCG, and its impact on the TST, the
vaccine is not recommended for general use.
Notes
 BCG vaccination induces
TST reactivity, which tends
to wane with time.
 The presence or size of TST
reactions after vaccination
does not predict the degree
of protection afforded.
RESEARCH AND INNOVATION
RESEARCH AND INNOVATION
 The diagnostic pipeline has expanded considerably in terms of
the number of tests, products or methods in development.
These include molecular tests for the detection of TB disease
and drug resistance, interferon-gamma release assays
(IGRAs) for the detection of TB infection, biomarker-based
assays for detection of TB disease, computer-aided detection
(CAD) for TB screening using digital chest radiography, and a
new class of aerosol-capture technologies for detection of TB
disease.
 Three new antigen-based skin tests for TB infection that
perform better than tuberculin skin tests (particularly in terms of
specificity) were evaluated and recommended by WHO in
2022.
RESEARCH AND INNOVATION
 There were 16 vaccine candidates in clinical trials by September
2022: four in Phase I, eight in Phase II and four in Phase III. They
included candidates to prevent TB infection and TB disease, and to
help improve the outcomes of treatment for TB disease.
 In September 2022, there were 26 drugs for the treatment of TB
disease in Phase I, Phase II or Phase III trials. These drugs comprise
17 new chemical entities, two drugs that have received accelerated
regulatory approval, one drug that was recently approved by the
United States (US) Food and Drug Administration under the limited
population pathway for antibacterial and antifungal drugs, and six
repurposed drugs. There are at least 22 clinical trials to evaluate
drugs and drug regimens for treatment of TB infection.
Cases
Case 1
 A 35-year-old man comes to the physician with a 2-month history of fever,
night sweats, and a cough productive of white sputum. He lives in New York
and is currently homeless. He uses intravenous heroin several times a week.
Three months ago he was admitted to the hospital for aspiration pneumonia.
His dad passed away at the age of 54 because of “a bad heart condition.” His
temperature is 38°C (100.4°F) and respirations are 22/min. Physical
examination shows coarse crackles in the left upper posterior lung field. An x-
ray of the chest shows a cavitary lesion with calcification in the left upper lobe.
Which of the following is the most likely source of his pulmonary findings?
1. Aspiration of oral flora
2. Exposure to contaminated hot water tanks
3. Reactivation of a latent infection
4. Embolization of a bacterial vegetation
Case 2
 A 46-year-old man who is a prisoner comes to the county jail urgent care clinic because of a 1-month
history of productive cough, malaise, and right-sided chest pain. During this time, he also has had
shortness of breath and a 6-kg (13-Ib) weight loss. Three weeks ago, he was treated with amoxicillin for
pneumonia but his symptoms did not improve. He worked as a construction worker for 15 years. He has
HIV infection and hypertension. Medications include lisinopril and antiretroviral therapy. His temperature is
38.4°C (101.1°F) and blood pressure is 142/88 mm Hg. Pulse oximetry on room air shows an oxygen
saturation of 91%. On pulmonary examination, scattered crackles are heard bilaterally; there is dullness to
percussion at the right lung base. Chest x-ray shows right-sided hilar lymphadenopathy, a fibrous, cavitary
lesion in the right lower lobe, and right-sided pleural effusion. CD4 count is 600 cells/mm3. Which of the
following is the most likely diagnosis?
1. Hodking lymphoma
2. Pulmonary sarcoidosis
3. Pneumocystis Jerovecii pneumonia
4. Primary tuberculosis
5. Mesothelioma
Case 3
 A 54-year-old woman comes to the physician because of lower back pain,
night sweats, and a 5-kg (11-lb) weight loss during the past 4 weeks. She has
rheumatoid arthritis treated with adalimumab. Her temperature is 38°C
(100.4°F). Physical examination shows tenderness over the T10 and L1
spinous processes. Passive extension of the right hip causes pain in the right
lower quadrant. The patient's symptoms are most likely caused by an
organism with which of the following virulence factors?
1. Polysaccharide capsule that prevents phagocytosis
2. Surface glycolipids that prevent phagolysosome fusion
3. Polypeptides that inactivate elongation factor 2
4. Proteins that bind to the Fc region of immunoglobulin G
5. Protease that cleaves immunoglobulin A
Case 4
 A previously healthy 67-year-old man comes to the physician for routine health
maintenance evaluation. He works at a community center and volunteers at a local
homeless shelter. A tuberculin skin test shows an induration of 14 mm. An x-ray of the
chest is normal. Treatment with an antimycobacterial drug is initiated. Two months later,
he has numbness and burning over both feet and an unsteady gait. Physical examination
shows decreased sensation to light touch extending from the soles of the feet to the mid-
shin bilaterally. This patient's condition is most likely to result in which of the following?
1. Impaired synthesis of neurotransmitters
2. Intracellular accumulation of sorbitol
3. Segmental demyelination of peripheral axons
4. Impaired breakdown of glucose to ATP
5. Degeneration of the dorsal columns
Case 5
 A 48-year-old man comes to the physician for a preplacement examination and tuberculosis test before
starting a volunteer job in a refugee clinic. He has had a mild cough, sore throat, and occasional
headaches for 1 week. He has hypertension and gastroesophageal reflux disease. He works as a
mechanic and has never traveled outside of the US. He lives alone and has had no known contact with
anyone infected with tuberculosis. He has smoked one pack of cigarettes daily for 27 years and does not
drink alcohol. Current medications include ramipril and esomeprazole. He is 180 cm (5 ft 11 in) tall and
weighs 89 kg (196 lb); BMI is 28 kg/m2. His temperature is 37.3°C (99.1°F), pulse is 81/min, respirations
are 17/min, and blood pressure is 145/75 mm Hg. Pulmonary examination shows no abnormalities.
Laboratory studies show a leukocyte count of 6200/mm3 and a hemoglobin concentration of 13.8 g/dL. A
rapid test for COVID-19 is negative. A tuberculin skin test (TST) shows an induration of 13 mm in diameter
after 50 hours. Which of the following is the most appropriate next step in management?
1. Chest x-ray
2. No further management is indicated
3. Interferon-y release assay
4. Repeat tuberculin skin test after 8-10 weeks
5. Isoniazid and rifapentine therapy
Case 6
 A 55-year-old woman comes to the physician because of fevers for 2 weeks.
She works as a nurse and recently returned from a charity work trip to India,
where she worked in a medically-underserved rural community. A tuberculin
skin test 3 months ago prior to her trip showed an induration of 3 mm. Physical
examination is unremarkable. An x-ray of the chest shows right-sided hilar
lymphadenopathy. A sputum culture shows acid-fast bacilli. Which of the
following immunologic processes most likely occurred first?
1. Transportation of bacterial peptides to regional lymph nodes
2. Formation of a nodular tubercle in the lung
3. Replication of bacteria within alveolar macrophages
4. Fusion of phagolysosomes in neutrophils
5. Production of interferon-gamma by T-helper cells
6. Migration of T-helper cells to the lungs
Case 7
 A 4-year-old boy is brought to the physician by his mother because she is worried that her
child may have tuberculosis. One week ago, the patient returned from a trip to Moldova,
where he visited his grandparents. The grandfather has had a cough during the past
weeks and received the diagnosis of active tuberculosis (TB) 2 days ago. The child feels
well. He has no history of serious illness and takes no medications. Immunizations are up-
to-date. His temperature is 36.8°C (98.2°F), pulse is 88/min, respirations are 20/min, and
blood pressure is 95/60 mm Hg. Physical examination and chest x-ray show no
abnormalities. A tuberculin skin test (TST) shows an induration of 3 mm in diameter after 2
days. Which of the following is the most appropriate next step in management?
1. Repeat tuberculin skin test in 1 week
2. No further management is indicated at this time
3. Perform interferon-gamma release assay
4. Initiate rifampin therapy
5. Initiate rifampin, isoniazid, pyrazinamide, and ethambutol therapy

More Related Content

Similar to Tuberculosis.pptx

Mycobacterium tuberculosis
Mycobacterium tuberculosisMycobacterium tuberculosis
Mycobacterium tuberculosis
hodmedicine
 
Tuberculosis
TuberculosisTuberculosis
Tuberculosis
Ngk Sharma
 
Pathophysiology tubercuslosis
Pathophysiology tubercuslosisPathophysiology tubercuslosis
Pathophysiology tubercuslosis
Nem kumar Jain
 
Pathophysiology of Tuberculosis and it's Complications
Pathophysiology of Tuberculosis and it's ComplicationsPathophysiology of Tuberculosis and it's Complications
Pathophysiology of Tuberculosis and it's Complications
chiranrudresh1
 
Tuberculosis lecture
Tuberculosis lecture Tuberculosis lecture
Tuberculosis lecture
PHARMA IQ EDUCATION
 
Mycobacterium tuberculosis
Mycobacterium tuberculosisMycobacterium tuberculosis
Mycobacterium tuberculosis
Suprakash Das
 
Mycobacteria slides for lecture
Mycobacteria slides for lectureMycobacteria slides for lecture
Mycobacteria slides for lectureBruno Mmassy
 
Mycobacteria slides for lecture
Mycobacteria slides for lectureMycobacteria slides for lecture
Mycobacteria slides for lectureBruno Mmassy
 
Immune Resopnse to Tuberculosis
Immune Resopnse to TuberculosisImmune Resopnse to Tuberculosis
Immune Resopnse to Tuberculosis
Mano Vina
 
Tuberculosis (tb)
Tuberculosis (tb)Tuberculosis (tb)
Tuberculosis (tb)
JAYANTHBM
 
Herd immunity
Herd immunityHerd immunity
Herd immunity
Mazin-Salim Almazini
 
5. Kuliah Infeksi TB - Dr. Reviono,dr., Sp.P(K) .pptx
5. Kuliah Infeksi TB - Dr. Reviono,dr., Sp.P(K) .pptx5. Kuliah Infeksi TB - Dr. Reviono,dr., Sp.P(K) .pptx
5. Kuliah Infeksi TB - Dr. Reviono,dr., Sp.P(K) .pptx
HakimAlhaady
 
Tuberculosis
TuberculosisTuberculosis
Tuberculosis
ZeelNaik2
 
Infectious Diseases.pptx
Infectious Diseases.pptxInfectious Diseases.pptx
Infectious Diseases.pptx
Sunil Kardani
 
tuberculosis .pptx
tuberculosis .pptxtuberculosis .pptx
tuberculosis .pptx
SaketKumar946153
 
tuberculosis (2).pptx
tuberculosis (2).pptxtuberculosis (2).pptx
tuberculosis (2).pptx
SaketKumar946153
 
9 tuberculosis tanweiping
9 tuberculosis tanweiping9 tuberculosis tanweiping
9 tuberculosis tanweipingSumit Prajapati
 
Tuberculosis in pediatrics
Tuberculosis in pediatricsTuberculosis in pediatrics
Tuberculosis in pediatrics
El Verlain
 
Tuberculosis
TuberculosisTuberculosis
Tuberculosis
Muhammad iqbal
 
TUBERCULOSIS- a complete Guide
TUBERCULOSIS- a complete GuideTUBERCULOSIS- a complete Guide
TUBERCULOSIS- a complete Guide
Abith Baburaj
 

Similar to Tuberculosis.pptx (20)

Mycobacterium tuberculosis
Mycobacterium tuberculosisMycobacterium tuberculosis
Mycobacterium tuberculosis
 
Tuberculosis
TuberculosisTuberculosis
Tuberculosis
 
Pathophysiology tubercuslosis
Pathophysiology tubercuslosisPathophysiology tubercuslosis
Pathophysiology tubercuslosis
 
Pathophysiology of Tuberculosis and it's Complications
Pathophysiology of Tuberculosis and it's ComplicationsPathophysiology of Tuberculosis and it's Complications
Pathophysiology of Tuberculosis and it's Complications
 
Tuberculosis lecture
Tuberculosis lecture Tuberculosis lecture
Tuberculosis lecture
 
Mycobacterium tuberculosis
Mycobacterium tuberculosisMycobacterium tuberculosis
Mycobacterium tuberculosis
 
Mycobacteria slides for lecture
Mycobacteria slides for lectureMycobacteria slides for lecture
Mycobacteria slides for lecture
 
Mycobacteria slides for lecture
Mycobacteria slides for lectureMycobacteria slides for lecture
Mycobacteria slides for lecture
 
Immune Resopnse to Tuberculosis
Immune Resopnse to TuberculosisImmune Resopnse to Tuberculosis
Immune Resopnse to Tuberculosis
 
Tuberculosis (tb)
Tuberculosis (tb)Tuberculosis (tb)
Tuberculosis (tb)
 
Herd immunity
Herd immunityHerd immunity
Herd immunity
 
5. Kuliah Infeksi TB - Dr. Reviono,dr., Sp.P(K) .pptx
5. Kuliah Infeksi TB - Dr. Reviono,dr., Sp.P(K) .pptx5. Kuliah Infeksi TB - Dr. Reviono,dr., Sp.P(K) .pptx
5. Kuliah Infeksi TB - Dr. Reviono,dr., Sp.P(K) .pptx
 
Tuberculosis
TuberculosisTuberculosis
Tuberculosis
 
Infectious Diseases.pptx
Infectious Diseases.pptxInfectious Diseases.pptx
Infectious Diseases.pptx
 
tuberculosis .pptx
tuberculosis .pptxtuberculosis .pptx
tuberculosis .pptx
 
tuberculosis (2).pptx
tuberculosis (2).pptxtuberculosis (2).pptx
tuberculosis (2).pptx
 
9 tuberculosis tanweiping
9 tuberculosis tanweiping9 tuberculosis tanweiping
9 tuberculosis tanweiping
 
Tuberculosis in pediatrics
Tuberculosis in pediatricsTuberculosis in pediatrics
Tuberculosis in pediatrics
 
Tuberculosis
TuberculosisTuberculosis
Tuberculosis
 
TUBERCULOSIS- a complete Guide
TUBERCULOSIS- a complete GuideTUBERCULOSIS- a complete Guide
TUBERCULOSIS- a complete Guide
 

More from Fuad952583

Antenatal care_3.ppt
Antenatal care_3.pptAntenatal care_3.ppt
Antenatal care_3.ppt
Fuad952583
 
Prevention of Sensory diseases.pptx
Prevention of Sensory diseases.pptxPrevention of Sensory diseases.pptx
Prevention of Sensory diseases.pptx
Fuad952583
 
4_5888527186473782561.ppt
4_5888527186473782561.ppt4_5888527186473782561.ppt
4_5888527186473782561.ppt
Fuad952583
 
General roule for prevention بعد الحذف(1).en.ar.pptx
General roule for prevention بعد الحذف(1).en.ar.pptxGeneral roule for prevention بعد الحذف(1).en.ar.pptx
General roule for prevention بعد الحذف(1).en.ar.pptx
Fuad952583
 
حساب درجات البيسيك.pdf
حساب درجات البيسيك.pdfحساب درجات البيسيك.pdf
حساب درجات البيسيك.pdf
Fuad952583
 
nevus .pdf
nevus .pdfnevus .pdf
nevus .pdf
Fuad952583
 
skin disorder .pdf
skin disorder .pdfskin disorder .pdf
skin disorder .pdf
Fuad952583
 
histology of Salivary Glands .pptx
histology of Salivary Glands .pptxhistology of Salivary Glands .pptx
histology of Salivary Glands .pptx
Fuad952583
 
Noor-Book.com ألغاز ومسابقات ثقافات فكاهات معلومات قصص رائعة محمد عدنان غنام...
Noor-Book.com  ألغاز ومسابقات ثقافات فكاهات معلومات قصص رائعة محمد عدنان غنام...Noor-Book.com  ألغاز ومسابقات ثقافات فكاهات معلومات قصص رائعة محمد عدنان غنام...
Noor-Book.com ألغاز ومسابقات ثقافات فكاهات معلومات قصص رائعة محمد عدنان غنام...
Fuad952583
 
Slides_13_Types_of_Tissues_Meyer.pdf
Slides_13_Types_of_Tissues_Meyer.pdfSlides_13_Types_of_Tissues_Meyer.pdf
Slides_13_Types_of_Tissues_Meyer.pdf
Fuad952583
 

More from Fuad952583 (10)

Antenatal care_3.ppt
Antenatal care_3.pptAntenatal care_3.ppt
Antenatal care_3.ppt
 
Prevention of Sensory diseases.pptx
Prevention of Sensory diseases.pptxPrevention of Sensory diseases.pptx
Prevention of Sensory diseases.pptx
 
4_5888527186473782561.ppt
4_5888527186473782561.ppt4_5888527186473782561.ppt
4_5888527186473782561.ppt
 
General roule for prevention بعد الحذف(1).en.ar.pptx
General roule for prevention بعد الحذف(1).en.ar.pptxGeneral roule for prevention بعد الحذف(1).en.ar.pptx
General roule for prevention بعد الحذف(1).en.ar.pptx
 
حساب درجات البيسيك.pdf
حساب درجات البيسيك.pdfحساب درجات البيسيك.pdf
حساب درجات البيسيك.pdf
 
nevus .pdf
nevus .pdfnevus .pdf
nevus .pdf
 
skin disorder .pdf
skin disorder .pdfskin disorder .pdf
skin disorder .pdf
 
histology of Salivary Glands .pptx
histology of Salivary Glands .pptxhistology of Salivary Glands .pptx
histology of Salivary Glands .pptx
 
Noor-Book.com ألغاز ومسابقات ثقافات فكاهات معلومات قصص رائعة محمد عدنان غنام...
Noor-Book.com  ألغاز ومسابقات ثقافات فكاهات معلومات قصص رائعة محمد عدنان غنام...Noor-Book.com  ألغاز ومسابقات ثقافات فكاهات معلومات قصص رائعة محمد عدنان غنام...
Noor-Book.com ألغاز ومسابقات ثقافات فكاهات معلومات قصص رائعة محمد عدنان غنام...
 
Slides_13_Types_of_Tissues_Meyer.pdf
Slides_13_Types_of_Tissues_Meyer.pdfSlides_13_Types_of_Tissues_Meyer.pdf
Slides_13_Types_of_Tissues_Meyer.pdf
 

Recently uploaded

Ocular injury ppt Upendra pal optometrist upums saifai etawah
Ocular injury  ppt  Upendra pal  optometrist upums saifai etawahOcular injury  ppt  Upendra pal  optometrist upums saifai etawah
Ocular injury ppt Upendra pal optometrist upums saifai etawah
pal078100
 
ARTIFICIAL INTELLIGENCE IN HEALTHCARE.pdf
ARTIFICIAL INTELLIGENCE IN  HEALTHCARE.pdfARTIFICIAL INTELLIGENCE IN  HEALTHCARE.pdf
ARTIFICIAL INTELLIGENCE IN HEALTHCARE.pdf
Anujkumaranit
 
Flu Vaccine Alert in Bangalore Karnataka
Flu Vaccine Alert in Bangalore KarnatakaFlu Vaccine Alert in Bangalore Karnataka
Flu Vaccine Alert in Bangalore Karnataka
addon Scans
 
Lung Cancer: Artificial Intelligence, Synergetics, Complex System Analysis, S...
Lung Cancer: Artificial Intelligence, Synergetics, Complex System Analysis, S...Lung Cancer: Artificial Intelligence, Synergetics, Complex System Analysis, S...
Lung Cancer: Artificial Intelligence, Synergetics, Complex System Analysis, S...
Oleg Kshivets
 
Couples presenting to the infertility clinic- Do they really have infertility...
Couples presenting to the infertility clinic- Do they really have infertility...Couples presenting to the infertility clinic- Do they really have infertility...
Couples presenting to the infertility clinic- Do they really have infertility...
Sujoy Dasgupta
 
POST OPERATIVE OLIGURIA and its management
POST OPERATIVE OLIGURIA and its managementPOST OPERATIVE OLIGURIA and its management
POST OPERATIVE OLIGURIA and its management
touseefaziz1
 
Charaka Samhita Sutra Sthana 9 Chapter khuddakachatuspadadhyaya
Charaka Samhita Sutra Sthana 9 Chapter khuddakachatuspadadhyayaCharaka Samhita Sutra Sthana 9 Chapter khuddakachatuspadadhyaya
Charaka Samhita Sutra Sthana 9 Chapter khuddakachatuspadadhyaya
Dr KHALID B.M
 
THOA 2.ppt Human Organ Transplantation Act
THOA 2.ppt Human Organ Transplantation ActTHOA 2.ppt Human Organ Transplantation Act
THOA 2.ppt Human Organ Transplantation Act
DrSathishMS1
 
Tom Selleck Health: A Comprehensive Look at the Iconic Actor’s Wellness Journey
Tom Selleck Health: A Comprehensive Look at the Iconic Actor’s Wellness JourneyTom Selleck Health: A Comprehensive Look at the Iconic Actor’s Wellness Journey
Tom Selleck Health: A Comprehensive Look at the Iconic Actor’s Wellness Journey
greendigital
 
Prix Galien International 2024 Forum Program
Prix Galien International 2024 Forum ProgramPrix Galien International 2024 Forum Program
Prix Galien International 2024 Forum Program
Levi Shapiro
 
TEST BANK for Operations Management, 14th Edition by William J. Stevenson, Ve...
TEST BANK for Operations Management, 14th Edition by William J. Stevenson, Ve...TEST BANK for Operations Management, 14th Edition by William J. Stevenson, Ve...
TEST BANK for Operations Management, 14th Edition by William J. Stevenson, Ve...
kevinkariuki227
 
Are There Any Natural Remedies To Treat Syphilis.pdf
Are There Any Natural Remedies To Treat Syphilis.pdfAre There Any Natural Remedies To Treat Syphilis.pdf
Are There Any Natural Remedies To Treat Syphilis.pdf
Little Cross Family Clinic
 
The POPPY STUDY (Preconception to post-partum cardiovascular function in prim...
The POPPY STUDY (Preconception to post-partum cardiovascular function in prim...The POPPY STUDY (Preconception to post-partum cardiovascular function in prim...
The POPPY STUDY (Preconception to post-partum cardiovascular function in prim...
Catherine Liao
 
ARTHROLOGY PPT NCISM SYLLABUS AYURVEDA STUDENTS
ARTHROLOGY PPT NCISM SYLLABUS AYURVEDA STUDENTSARTHROLOGY PPT NCISM SYLLABUS AYURVEDA STUDENTS
ARTHROLOGY PPT NCISM SYLLABUS AYURVEDA STUDENTS
Dr. Vinay Pareek
 
Surgical Site Infections, pathophysiology, and prevention.pptx
Surgical Site Infections, pathophysiology, and prevention.pptxSurgical Site Infections, pathophysiology, and prevention.pptx
Surgical Site Infections, pathophysiology, and prevention.pptx
jval Landero
 
The hemodynamic and autonomic determinants of elevated blood pressure in obes...
The hemodynamic and autonomic determinants of elevated blood pressure in obes...The hemodynamic and autonomic determinants of elevated blood pressure in obes...
The hemodynamic and autonomic determinants of elevated blood pressure in obes...
Catherine Liao
 
Report Back from SGO 2024: What’s the Latest in Cervical Cancer?
Report Back from SGO 2024: What’s the Latest in Cervical Cancer?Report Back from SGO 2024: What’s the Latest in Cervical Cancer?
Report Back from SGO 2024: What’s the Latest in Cervical Cancer?
bkling
 
Surat @ℂall @Girls ꧁❤8527049040❤꧂@ℂall @Girls Service Vip Top Model Safe
Surat @ℂall @Girls ꧁❤8527049040❤꧂@ℂall @Girls Service Vip Top Model SafeSurat @ℂall @Girls ꧁❤8527049040❤꧂@ℂall @Girls Service Vip Top Model Safe
Surat @ℂall @Girls ꧁❤8527049040❤꧂@ℂall @Girls Service Vip Top Model Safe
Savita Shen $i11
 
MANAGEMENT OF ATRIOVENTRICULAR CONDUCTION BLOCK.pdf
MANAGEMENT OF ATRIOVENTRICULAR CONDUCTION BLOCK.pdfMANAGEMENT OF ATRIOVENTRICULAR CONDUCTION BLOCK.pdf
MANAGEMENT OF ATRIOVENTRICULAR CONDUCTION BLOCK.pdf
Jim Jacob Roy
 
Physiology of Chemical Sensation of smell.pdf
Physiology of Chemical Sensation of smell.pdfPhysiology of Chemical Sensation of smell.pdf
Physiology of Chemical Sensation of smell.pdf
MedicoseAcademics
 

Recently uploaded (20)

Ocular injury ppt Upendra pal optometrist upums saifai etawah
Ocular injury  ppt  Upendra pal  optometrist upums saifai etawahOcular injury  ppt  Upendra pal  optometrist upums saifai etawah
Ocular injury ppt Upendra pal optometrist upums saifai etawah
 
ARTIFICIAL INTELLIGENCE IN HEALTHCARE.pdf
ARTIFICIAL INTELLIGENCE IN  HEALTHCARE.pdfARTIFICIAL INTELLIGENCE IN  HEALTHCARE.pdf
ARTIFICIAL INTELLIGENCE IN HEALTHCARE.pdf
 
Flu Vaccine Alert in Bangalore Karnataka
Flu Vaccine Alert in Bangalore KarnatakaFlu Vaccine Alert in Bangalore Karnataka
Flu Vaccine Alert in Bangalore Karnataka
 
Lung Cancer: Artificial Intelligence, Synergetics, Complex System Analysis, S...
Lung Cancer: Artificial Intelligence, Synergetics, Complex System Analysis, S...Lung Cancer: Artificial Intelligence, Synergetics, Complex System Analysis, S...
Lung Cancer: Artificial Intelligence, Synergetics, Complex System Analysis, S...
 
Couples presenting to the infertility clinic- Do they really have infertility...
Couples presenting to the infertility clinic- Do they really have infertility...Couples presenting to the infertility clinic- Do they really have infertility...
Couples presenting to the infertility clinic- Do they really have infertility...
 
POST OPERATIVE OLIGURIA and its management
POST OPERATIVE OLIGURIA and its managementPOST OPERATIVE OLIGURIA and its management
POST OPERATIVE OLIGURIA and its management
 
Charaka Samhita Sutra Sthana 9 Chapter khuddakachatuspadadhyaya
Charaka Samhita Sutra Sthana 9 Chapter khuddakachatuspadadhyayaCharaka Samhita Sutra Sthana 9 Chapter khuddakachatuspadadhyaya
Charaka Samhita Sutra Sthana 9 Chapter khuddakachatuspadadhyaya
 
THOA 2.ppt Human Organ Transplantation Act
THOA 2.ppt Human Organ Transplantation ActTHOA 2.ppt Human Organ Transplantation Act
THOA 2.ppt Human Organ Transplantation Act
 
Tom Selleck Health: A Comprehensive Look at the Iconic Actor’s Wellness Journey
Tom Selleck Health: A Comprehensive Look at the Iconic Actor’s Wellness JourneyTom Selleck Health: A Comprehensive Look at the Iconic Actor’s Wellness Journey
Tom Selleck Health: A Comprehensive Look at the Iconic Actor’s Wellness Journey
 
Prix Galien International 2024 Forum Program
Prix Galien International 2024 Forum ProgramPrix Galien International 2024 Forum Program
Prix Galien International 2024 Forum Program
 
TEST BANK for Operations Management, 14th Edition by William J. Stevenson, Ve...
TEST BANK for Operations Management, 14th Edition by William J. Stevenson, Ve...TEST BANK for Operations Management, 14th Edition by William J. Stevenson, Ve...
TEST BANK for Operations Management, 14th Edition by William J. Stevenson, Ve...
 
Are There Any Natural Remedies To Treat Syphilis.pdf
Are There Any Natural Remedies To Treat Syphilis.pdfAre There Any Natural Remedies To Treat Syphilis.pdf
Are There Any Natural Remedies To Treat Syphilis.pdf
 
The POPPY STUDY (Preconception to post-partum cardiovascular function in prim...
The POPPY STUDY (Preconception to post-partum cardiovascular function in prim...The POPPY STUDY (Preconception to post-partum cardiovascular function in prim...
The POPPY STUDY (Preconception to post-partum cardiovascular function in prim...
 
ARTHROLOGY PPT NCISM SYLLABUS AYURVEDA STUDENTS
ARTHROLOGY PPT NCISM SYLLABUS AYURVEDA STUDENTSARTHROLOGY PPT NCISM SYLLABUS AYURVEDA STUDENTS
ARTHROLOGY PPT NCISM SYLLABUS AYURVEDA STUDENTS
 
Surgical Site Infections, pathophysiology, and prevention.pptx
Surgical Site Infections, pathophysiology, and prevention.pptxSurgical Site Infections, pathophysiology, and prevention.pptx
Surgical Site Infections, pathophysiology, and prevention.pptx
 
The hemodynamic and autonomic determinants of elevated blood pressure in obes...
The hemodynamic and autonomic determinants of elevated blood pressure in obes...The hemodynamic and autonomic determinants of elevated blood pressure in obes...
The hemodynamic and autonomic determinants of elevated blood pressure in obes...
 
Report Back from SGO 2024: What’s the Latest in Cervical Cancer?
Report Back from SGO 2024: What’s the Latest in Cervical Cancer?Report Back from SGO 2024: What’s the Latest in Cervical Cancer?
Report Back from SGO 2024: What’s the Latest in Cervical Cancer?
 
Surat @ℂall @Girls ꧁❤8527049040❤꧂@ℂall @Girls Service Vip Top Model Safe
Surat @ℂall @Girls ꧁❤8527049040❤꧂@ℂall @Girls Service Vip Top Model SafeSurat @ℂall @Girls ꧁❤8527049040❤꧂@ℂall @Girls Service Vip Top Model Safe
Surat @ℂall @Girls ꧁❤8527049040❤꧂@ℂall @Girls Service Vip Top Model Safe
 
MANAGEMENT OF ATRIOVENTRICULAR CONDUCTION BLOCK.pdf
MANAGEMENT OF ATRIOVENTRICULAR CONDUCTION BLOCK.pdfMANAGEMENT OF ATRIOVENTRICULAR CONDUCTION BLOCK.pdf
MANAGEMENT OF ATRIOVENTRICULAR CONDUCTION BLOCK.pdf
 
Physiology of Chemical Sensation of smell.pdf
Physiology of Chemical Sensation of smell.pdfPhysiology of Chemical Sensation of smell.pdf
Physiology of Chemical Sensation of smell.pdf
 

Tuberculosis.pptx

  • 2. Historical background  TB is an ancient disease.  Signs of skeletal TB (Pott disease) have been found in remains from Europe from Neolithic times (8000 BCE), ancient Egypt (1000 BCE),  German physician Robert Koch discovered and isolated M tuberculosis in 1882.  by the Industrial Revolution in Europe (1750), it was responsible for more than 25% of adult deaths.
  • 4. Epidemiology  TB SITUATION AND RESPONSE  Tuberculosis (TB) is contagious and airborne.  TB is the second leading infectious killer after COVID-19 and the 13th leading cause of death worldwide. It is also the leading killer of people with HIV and a major cause of deaths related to antimicrobial resistance  One-fourth of the world's population has latent TB.
  • 5.
  • 6.
  • 7.
  • 8.
  • 9.
  • 10. Etiology  Mycobacterium species that cause tuberculosis are collectively known as the Mycobacterium tuberculosis complex, which includes:  Mycobacterium tuberculosis  Mode of transmission: spread via aerosol droplet nuclei  Reservoir: predominantly humans  Disease: all forms of tuberculosis  Mycobacterium bovis  Mode of transmission: predominantly via ingestion of contaminated cow's milk  Reservoir: predominantly cattle  Disease: gastrointestinal tuberculosis in humans  Mycobacterium africanum.  Mycobacterium microti.
  • 11. Etiology  Mycobacteria other than tuberculosis (MOTT) most often cause disease in individuals with weakened immune systems  Mycobacterium avium and M. intracellulare are the more common MOTT sometimes seen in patients co-infected with HIV
  • 12. Etiology  Features of Mycobacterium tuberculosis  Type: facultative intracellular rod-shaped bacteria  Gram stain: does not stain well  Special stains 1. Ziehl-Neelsen stain: acid-fast bacilli appear pink 2. Auramine-rhodamine stain • Acid-fast bacilli appear reddish-yellow on fluorescence microscopy. • Used as a screening tool because of high sensitivity and low cost  Culture mediums for growth 1. Löwenstein Jensen medium 2. Middlebrook medium 3. Rapid automated broth culture
  • 13.
  • 14. Types of tuberculosis  Drug-resistant tuberculosis  Definition: a form of TB that is resistant to one or more antitubercular agents  Types 1. Rifampin-resistant tuberculosis (RR-TB): resistance to rifampin with or without resistance to other antitubercular drugs 2. Monoresistant TB: resistance to one of the first-line antitubercular drugs 3. Multidrug-resistant tuberculosis (MDR-TB): resistance to both isoniazid and rifampin 4. Extensively drug-resistant tuberculosis (XDR-TB): resistance to any single fluoroquinolone and at least one of the second-line injectable drugs (e.g., amikacin, kanamycin, or capreomycin) in addition to MDR-TB  Causes 1. Incorrect drug combination therapy 2. Inadequate duration or dosage of drug therapy 3. Poor treatment adherence 4. Poor quality of drugs 5. Close contact with an individual with drug-resistant TB
  • 15. FROM EXPOSURE TO INFECTION  M. tuberculosis is most commonly transmitted from a person with infectious pulmonary TB by droplet nuclei containing M. tuberculosis bacteria, which are aerosolized by coughing, sneezing, or speaking. The tiny droplets dry rapidly; the smallest (<5–10 μm in diameter) may remain suspended in the air for several hours and may reach the terminal air passages when inhaled. There may be as many as 3000 infectious nuclei per cough.  Other routes of transmission of tubercle bacilli (e.g., through the skin or the placenta) are uncommon and of no epidemiologic significance.  The risk of transmission and of subsequent acquisition of M. tuberculosis infection is determined mainly by exogenous factors, although endogenous factors may also play a role. The probability of contact with a person who has an infectious form of TB, the intimacy and duration of that contact, the degree of infectiousness of the case, and the shared environment in which the contact takes place are all important determinants of the likelihood of transmission.
  • 16. FROM EXPOSURE TO INFECTION  Several studies of close-contact situations have clearly demonstrated that TB patients whose sputum contains AFB visible by microscopy (sputum smear–positive cases) are the most likely to transmit the infection. The most infectious patients have cavitary pulmonary disease or, much less commonly, laryngeal TB and produce sputum containing as many as 105–107 AFB/mL.  Patients with sputum smear–negative/culture-positive TB are less infectious, although they have been responsible for up to 20% of transmission in some studies in the United States.
  • 17.  the majority of inhaled bacilli are trapped in the upper airways and expelled by ciliated mucosal cells, a fraction (usually <10%) reach the alveoli, a unique immunoregulatory environment.
  • 18.
  • 19. Pathophysiology 1. Primary tuberculosis  Innate immune response  Exposure to M. tuberculosis 1. Individuals with M. tuberculosis infection disperse droplet nuclei that contain bacilli via sneezing or coughing. 2. Inhaled droplet nuclei reach the terminal alveoli and are taken up by the alveolar macrophages.  Entry into macrophages 1. Mycobacterial cell wall contains pathogen-associated molecular patterns (PAMPs) such as lipoarabinomannan and lipomannan. 2. Alveolar macrophages recognize M. tuberculosis PAMPs via toll-like receptors (TLRs) 3. Activation of TLRs leads to the production of proinflammatory cytokines (e.g., IL-1, IL-12, TNF-α) and phagocytosis of mycobacteria.
  • 20. Pathophysiology  Replication within macrophages  Typically, phagocytosed organisms reside within a phagosome to undergo intracellular killing via the following steps: 1. Phagosome maturation: acidification using a proton pump system 2. Fusion of phagosome and lysosome: mediated by increased intracellular calcium levels 3. Killing of bacteria by reactive oxygen species (ROS), reactive nitrogen intermediates (RNI), and lysosomal enzymes  M. tuberculosis survives within macrophages because of the inhibition of both phagosome maturation and phagolysosome fusion
  • 21. Pathophysiology  Virulence factors involved include:  Cord factor: This is a surface glycolipid that causes the bacterium to grow in a serpentine or cord-like pattern. It inhibits the migration of neutrophils and induces the release of TNF-α, a pro-inflammatory cytokine. TNF-α stimulates activated macrophages to form granulomas, which are organized clusters of immune cells that help contain the infection.  Sulfatides: These are surface glycolipids that inhibit the fusion of the phagosome with lysosomes, preventing the formation of a fully functional phagolysosome.
  • 22. Pathophysiology  Lipoarabinomannan: This is a lipoglycan, a complex molecule composed of lipids and sugars, that is found on the surface of M. tuberculosis. It induces the release of TNF-α from macrophages, contributing to the inflammatory response. Additionally, lipoarabinomannan can scavenge reactive oxygen species (ROS), which are toxic molecules produced by immune cells to destroy bacteria.  Catalase-peroxidase: This enzyme is produced by M. tuberculosis and helps the bacterium survive within the macrophage by catalyzing the destruction of ROS and hydrogen peroxide (H2O2). ROS and H2O2 are produced by immune cells as part of their defense mechanism against bacterial infections.
  • 23. Pathophysiology  Macrophage lysis and release of bacteria 1. Following replication in the alveolar macrophages, the released bacteria attack uninfected macrophages to spread infection. 2. Dendritic cells migrate to the site of infection and process mycobacterial antigens. 3. Some bacteria enter the bloodstream, causing bacteremia and seeding multiple organs.
  • 24. Pathophysiology  Cellular immune response  Th1 cell activation 1. Dendritic cells present mycobacterial antigens complexed with MHC 2 to naive T cells 2. Activated CD4+ T cells migrate to the focus of infection (type IV HSR).  Macrophage activation and bacterial killing • Activated CD4+ T cells release IFN-γ • IFN-γ acts on macrophages to enable bacterial killing via the following mechanisms: 1. Promotion of phagosome maturation 2. Enhanced RNI production 3. Autophagy
  • 25. Pathophysiology  Granulomatous inflammation and tissue destruction 1. IFN-γ-activated macrophages secrete TNF-α. 2. TNF-α promotes the aggregation of macrophages and T cells to form granulomas, affecting the lungs and regional lymph nodes 3. Destruction of M. tuberculosis-infected macrophages causes central caseous necrosis and tissue damage. 4. Granuloma limits the spread of infection. 5. Ghon focus: a granuloma typically located in the middle/lower lung lobes. 6. Ghon complex: formed by the Ghon focus, regional lymph node, and the linking lymphatic vessels
  • 26.
  • 27.
  • 28. Pathophysiology  Disease progression  Sufficient immune response  Most of the bacteria are killed.  Some bacteria may persist, causing LTBI.  The granulomas in the Ghon complex undergo fibrosis and calcification to form the Ranke complex.  Deficient immune response (e.g., HIV, malnutrition)  Failure of granulomas to limit infection  Progressive primary TB causing progressive lung disease, bacteremia, and miliary TB
  • 29.
  • 30. Pathophysiology 2. Secondary tuberculosis  Latent TB: Dynamic equilibrium is maintained between the host immune response and M. tuberculosis.  Reactivation of disease: due to weakening of immune response (e.g., resulting from HIV, TNF-α inhibitor therapy)  Disease progression  Caseating granulomas with central necrosis and Langhans giant cells are characteristic features.  Usually affects the upper lobes of the lungs because of higher oxygen tension  Can also affect other organs (due to seeding of organs in primary tuberculosis)  Prior sensitization to mycobacterial antigens results in a stronger inflammatory response, causing extensive tissue destruction, cavitation, and scarring.
  • 31.
  • 32. Clinical features  Pulmonary tuberculosis  Systemic 1. Low-grade fever with night sweats 2. Weight loss (often severe), anorexia 3. Decreased appetite 4. Malaise, weakness  Pulmonary 1. Non-productive cough 2. Symptoms of progression: productive cough with purulent sputum, hemoptysis 3. Shortness of breath 4. Pleuritic chest pain
  • 33.
  • 34.
  • 35. Clinical examination  Findings are nonspecific.  General: pallor, clubbing (advanced disease), generalized wasting  Chest examination: Findings vary depending on the type and degree of pulmonary involvement. 1. Consolidation: dullness on percussion, crackles, diminished breath sounds 2. Cavitation: hyperresonance on percussion, amphoric breath sounds 3. Bronchial obstruction: rhonchi 4. Pleural effusion and/or empyema: dullness on percussion, diminished breath sounds  Other possible findings: erythema nodosum, phlyctenular conjunctivitis
  • 36.
  • 38. Diagnostics  Microbiological studies  Confirmation of the presence of tuberculosis bacilli in different samples is done by direct visualization, positive culture, or by detecting genetic material. Samples used for testing include: 1. Sputum samples (most common): Obtain ≥ 3 samples separated by 8 to 24-hour intervals (should include at least one early morning sample). 2. Induced sputum 3. Gastric lavage 4. Bronchoalveolar lavage 5. Extrapulmonary TB suspected: Fluid specimens or tissue samples should be sent for cell count, chemistries, AFB smear microscopy, and NAAT
  • 39.
  • 40.
  • 41.
  • 42.
  • 43.
  • 44.
  • 45. Diagnostics  Drug susceptibility testing  Standard culture: used to assess for drug susceptibility in most patients; results are available after several weeks.  Rapid molecular testing : for select patients at high risk of resistant tuberculosis 1. History of previously treated tuberculosis 2. Contact with individuals with MDR-TB 3. HIV infection 4. Patients born or having lived for ≥ 1 year in a country with moderate to high TB incidence or high MDR-TB prevalence
  • 46.
  • 47.
  • 48.
  • 49.
  • 50.
  • 51.
  • 52.
  • 53.
  • 54.
  • 55.
  • 56. HIV and TB coinfection diagnosis  Diagnosing TB in patients with HIV coinfection is challenging because these individuals often have a negative AFB smear and commonly have atypical imaging findings resulting from paucibacillary disease due to a reduced immune response. Different measures can be used to increase diagnostic sensitivity.  Advanced testing in patients with HIV (not widely available) 1. For most patients: fluorescent AFB smear microscopy , specialized culture mediums 2. For patients with severe disease and those with a CD4 count < 100 cells/mm3: lateral flow urine lipoarabinomannan assay>>>>>A nonsputum-based diagnostic test used to detect mycobacterial antigens in urine. Mainly used for HIV patients with severe disease and/or those with a CD4 count < 100 cells/mm³ for a rapid point-of- care diagnosis.
  • 57. TREATMENT  Chemotherapy for TB became possible with the discovery of streptomycin in 1943.
  • 58. Treatment  General principles  Goals 1. Reduction of disease severity and risk of transmission 2. Eradication of all bacilli to achieve sustained cure without relapse following completion of treatment 3. Prevention of drug resistance during therapy  Infection control measures 1. Case notification: mandatory reporting to local health department 2. Airborne precautions: should include a surgical mask for the patient and adequate personal protective equipment (including respirators) for medical staff and caregivers 3. Contact tracing.
  • 59. Treatment  Pretreatment evaluation  Clinical assessment 1. Nutritional assessment 2. Symptom review 3. Eye exam: visual acuity and color discrimination  Microbiology: sputum smear microscopy and culture, drug susceptibility testing  Imaging: chest x-ray or other chest imaging  Laboratory studies 1. All patients: liver chemistries, platelet count, creatinine, HIV screening 2. Depending on risk: hepatitis B and hepatitis C screening, diabetes screen
  • 60. 1-Treatment of drug-susceptible TB  Intensive phase: 2 months of rifampin PLUS isoniazid, pyrazinamide, and ethambutol  The intensive phase allows for an increased antibiotic serum concentration, optimizes the killing of bacteria, and minimizes the risk of developing antibiotic resistance.  Continuation phase: 4 months of rifampin PLUS isoniazid  The continuation phase reduces the risk of relapse. If an individual has both cavitating disease and a positive sputum smear microscopy or culture at the end of the intensive phase of treatment, the continuation phase can be extended by three months (a total of seven months of continuation).  Adjuvant treatment: Pyridoxine to prevent vitamin B6 deficiency resulting from isoniazid (promotes pyridoxine excretion) for all individuals at risk of neuropathy
  • 61. 1-Treatment of drug-susceptible TB  Standard regimens are divided into an intensive (bactericidal) phase and a continuation (sterilizing) phase. During the intensive phase, the majority of tubercle bacilli are killed, symptoms resolve, and usually the patient becomes noninfectious. The continuation phase is required to eliminate persisting mycobacteria and prevent relapse.
  • 62. Directly observed therapy  Directly observed therapy (DOT; standard practice): Health care personnel observe the patient taking the medication.  Associated with increased treatment success, medication adherence, and prompt recognition of medication side effects  Restart full treatment if there is an interruption of ≥ 14 days during the intensive phase.
  • 63.
  • 64.
  • 65.
  • 66.
  • 67.
  • 68. 2-Treatment of MDR-TB  Drug-resistant TB may be either primary or acquired. In primary drug resistance, the patient is infected from the start by a drugresistant strain. Acquired resistance develops in the infecting strain during treatment.  Because there is no cross-resistance among the commonly used classes of drugs, the probability that a strain will be resistant to two drug classes is the product of the probabilities of resistance to each drug class and thus is low.
  • 69. 2-Treatment of MDR-TB  MDR-TB, in which bacilli are resistant to (at least) isoniazid and rifampin, is more difficult to manage than is disease caused by drug- susceptible organisms because these two bactericidal drugs are the most potent first-line agents available and because associated resistance to other first-line drugs as well (e.g., ethambutol) is not uncommon.  As a result, two main approaches are now recommended by the WHO to treat MDR/RR-TB: 1. an individualized longer regimen of 18–20 months’ duration consisting of an optimal combination of oral drugs chosen according to a rational approach and using the WHO priority grouping of medicines 2. a shorter, all-oral, bedaquiline-containing regimen of 9–12 months’ duration
  • 70. 2-Treatment of MDR-TB 1. Longer MDR-TB Regimen In MDR/RR-TB patients where infecting strains have or are presumed to have additional resistance (e.g., resistance to the fluoroquinolones), in those who have severe pulmonary or extrapulmonary disease, or those who have been treated previously with second-line drugs, a longer regimen is recommended. 2. Shorter MDR-TB Regimen In MDR/RR-TB patients with no extensive pulmonary disease or severe extrapulmonary disease, without history of previous treatment with second-line drugs, and whose infecting strains are not resistant to the fluoroquinolones, a shorter, all-oral, bedaquiline-containing regimen is recommended.
  • 71.
  • 72.
  • 73. Monitoring  Monthly follow-ups are recommended in all patients receiving antituberculosis treatment for active TB or LTBI.  All patients 1. Assess adherence to therapy. 2. Perform a symptom review and screen for clinical features of active TB. 3. Monitor for side effects.  Patients with active TB 1. Sputum smear microscopy and culture( Obtain monthly until two consecutive specimens are negative) 2. Weight and vision assessment(Vision assessment needs to be performed monthly for at least the first three months.) 3. Laboratory studies: If indicated, obtain liver chemistries, platelet count, and creatinine
  • 74. Monitoring  With the recommended 6-month standard first-line regimen, >80% of drug-susceptible TB patients will have negative sputum cultures at the end of the second month of treatment. By the end of the third month, the sputum of virtually all patients should be culture negative.  patients with cavitary disease in whom sputum culture conversion does not occur by 2 months require immediate testing or retesting for drug resistance. When a patient’s sputum cultures or smears remain positive at ≥3 months despite good adherence, treatment failure caused by drug resistance is likely.  A sputum specimen should be collected at the end of treatment to document cure  Bacteriologic monitoring of patients with extrapulmonary TB is more difficult and often is not feasible. In these cases, the response to treatment must be assessed clinically with the help of medical imaging.
  • 75. Definitions  New: a patient who has never had treatment for TB or who has take anti-tuberculosis drugs for less than 1 month.  Relapse: a patient previously treated for TB, who has been declared cured or treatment completed, and is diagnosed with bacteriologically positive (smear or culture) TB.  Treatment after failure: a patient who is started on a re-treatment regimen after having failed previous treatment.  Treatment after default: a patient who returns to treatment, positive bacteriologically, following interruption of treatment for 2 months or more.  Transfer in: a patient who has been transferred from another TB register to continue treatment.  Others: all cases that do not fit the above definitions. This group includes chronic case, a patient who is sputum-positive at the end of a re-treatment regimen.
  • 76. NOTE Most initially infectious patients become noninfectious within 2 weeks of starting effective treatment, and many become noninfectious within several days.
  • 78. Description  A state of constant immune response stimulation due to M. tuberculosis antigens, with no signs of active TB infection  Individuals with latent TB are asymptomatic and not contagious.
  • 79. Diagnosis of LTBI  The decision of whether to test an individual should be carefully considered based upon:- 1. the likelihood of someone having LTBI. 2. the likelihood of progression of LTBI to active TB. 3. the potential benefit of therapy.
  • 80. Diagnosis of LTBI  Individuals with a high likelihood of LTBI 1. Known contact with AFB smear-positive individual 2. Individuals born in or former residents of countries with a high TB burden (> 20 cases per 100,000 population) 3. People who live or work in high-risk settings (e.g., correctional facilities, long-term care facilities or nursing homes, homeless shelters) 4. Health-care workers who care for patients at increased risk for TB disease 5. Children (< 18 years of age) exposed to adults at increased risk for tuberculosis infection
  • 81. Diagnosis of LTBI  Individuals with a high likelihood of progression from LTBI to active TB  HIV infection  Immunosuppressive therapy, including:  Corticosteroid therapy  TNF-α inhibitor therapy (e.g., infliximab)  Others: methotrexate, cyclosporine; drugs used for organ transplant and stem cell transplant recipients  Silicosis
  • 82. Diagnosis of LTBI  Age < 5 years  Prior TB infection  Smoking  Heavy alcohol consumption  Malnutrition, low body mass index  Advanced age  Malignancy  Gastrectomy or jejunoileal bypass
  • 83.
  • 84. Interpretation of results  IGRA 1. Positive: TB infection is likely 2. Negative: TB infection is unlikely, but cannot be excluded 3. Indeterminate: can occur in immunosuppressed states, and a repeat IGRA or TST can be useful  TST: Depending on patient characteristics, a TST can be positive with an induration ≥ 5 mm, ≥ 10 mm, or ≥ 15 mm.  For healthy individuals with no risk factors, an induration < 15 mm is considered negative for TB.
  • 85.
  • 86.
  • 87.
  • 88. Treatment 1. Pharmacological therapy: indicated for patients with positive IGRA or TST after active TB has been excluded 2. Select a regimen based on the length of treatment, drug tolerance, pharmacological interactions, and HIV status.
  • 89. The primary goal of the treatment of latent TB is to prevent reactivation to active TB. NOTE
  • 90.
  • 92.
  • 93.
  • 94. Tuberculous lymphadenitis  Pathophysiology 1. Hematogenous dissemination following primary TB 2. Local extension of infection from the affected organ  Sites of lymph node involvement 1. Cervical 2. Inguinal 3. Axillary
  • 95. Tuberculous lymphadenitis  Clinical features  Constitutional symptoms  Lymphadenopathy: Findings vary depending on the stage of lymphadenitis. 1. Lymphadenitis: firm, mobile, and discrete lymph nodes 2. Periadenitis: rubbery and fixed lymph nodes 3. Cold abscess: soft and fluctuant lymph nodes 4. Draining sinus tract  Diagnostics 1. Ultrasound or CT: matted lymph nodes with a necrotic center 2. FNAC: acid-fast staining, PCR, and culture 3. Lymph node biopsy: histopathology  Treatment: See “Treatment of active TB.”
  • 96.
  • 97.
  • 98.
  • 99. Tuberculous pleurisy  Pathophysiology 1. Direct spread of infection from the lungs 2. Hematogenous spread 3. Delayed hypersensitivity reaction in the pleural space due to mycobacterial antigen  Clinical features 1. Constitutional symptoms 2. Nonproductive cough 3. Intensive (pleuritic) chest pain 4. Dyspnea
  • 100. Tuberculous pleurisy  Diagnostics  Imaging 1. Chest x-ray: unilateral pleural effusion 2. Chest ultrasound or CT: loculated effusion with septations; pleural thickening  Thoracentesis: pleural fluid analysis 1. Exudative type (Light criteria) 2. pH: < 7.4 3. Glucose: < 60 mg/dL 4. ↑ Adenosine deaminase 5. Rich in lymphocytes 6. Microbiology: acid-fast staining, PCR, and culture  Sputum smear examination: acid-fast staining 1. Treatment  Therapeutic thoracentesis for symptom relief
  • 101. Pericardial tuberculosis  Pathophysiology 1. Hematogenous spread from the site of primary infection 2. Retrograde lymphatic spread from hilar lymph nodes  Stages 1. Stage 1: fibrinous exudate containing neutrophils and mycobacteria 2. Stage 2: serosanguineous effusion with lymphocytes 3. Stage 3: absorption of effusion and caseating granulomas 4. Stage 4: scarring with calcification
  • 102. Pericardial tuberculosis  Types 1. Pericardial effusion (rarely, cardiac tamponade) 2. Constrictive pericarditis 3. Effusive-constrictive pericarditis  Clinical features 1. Constitutional symptoms 2. Chest pain 3. Cough 4. Dyspnea
  • 103. Pericardial tuberculosis  Physical examination findings: See “Clinical features” in “Pericardial effusion and cardiac tamponade” and “Pericarditis.”  Diagnostics  ECG: nonspecific ST-T changes  Imaging 1. Chest x-ray: enlarged cardiac silhouette or calcification 2. Echocardiography: effusion or diminished wall motility  Pericardiocentesis 1. Analysis: exudative, blood-stained 2. Microbiology: acid-fast staining, PCR, and culture 3. ↑ Adenosine deaminase  Treatment 1. See “Treatment of active TB.” 2. Therapeutic pericardiocentesis
  • 104.
  • 105.
  • 106.
  • 107. Miliary tuberculosis  Definition: a form of TB with multiorgan involvement that manifests with granulomatous lesions (resembling millets) as a result of lymphohematogenous dissemination of bacilli from a pulmonary or extrapulmonary source  Epidemiology: ∼ 20% of extrapulmonary TB cases  Pathophysiology: primary infection or reactivation  Common sites of involvement: lungs, spleen, liver, lymph nodes, adrenals, meninges, vertebrae (Pott disease), joints and long bones, and choroid  Clinical features: mostly nonspecific 1. Cough and dyspnea 2. Constitutional symptoms 3. Lymphadenopathy 4. Hepatosplenomegaly 5. Tuberculosis miliaria cutis disseminata
  • 108. Miliary tuberculosis  Diagnostics: requires a high index of clinical suspicion  Fundoscopy: choroid tubercles  Microbiology: sputum, body fluids, and tissue 1. Acid-fast staining 2. PCR 3. Culture  Blood 1. ↑ ESR and CRP 2. CBC: anemia, leukopenia 3. LFT: elevated transaminases and bilirubin 4. Electrolytes: hyponatremia, hypercalcemia  TST: negative
  • 109. Miliary tuberculosis  Imaging 1. Chest x-ray: multiple small nodules (< 2 mm) with an appearance resembling millet seeds 2. Ultrasound: pleural effusion, ascites, hepatic or splenic lesions 3. CT: lung nodules, enlarged hilar lymph nodes 4. Brain MRI: meningeal and/or basal cistern enhancement, hydrocephalus, tuberculoma 5. Echocardiography: pericardial effusion  Treatment: See “Treatment of active TB.”
  • 110.
  • 111. Gastrointestinal tuberculosis  Pathophysiology 1. Ingestion of infected milk or sputum 2. Hematogenous spread resulting from primary pulmonary TB 3. Contiguous spread via affected lymph nodes  Sites of involvement: See “Types of gastrointestinal TB” below.
  • 112.
  • 113. Treatment 1. Surgical: for intestinal obstruction, perforation, fistula, abscess 2. Medical: See “Treatment of active TB.”
  • 114. Renal and urologic TB  Pathophysiology 1. Hematogenous spread of infection to the kidneys as a result of primary pulmonary TB that is reactivated in immunosuppressed states 2. Ureters and bladder are infected when bacteria are excreted in urine.  Common sites of involvement 1. Kidneys 2. Ureters 3. Bladder
  • 115. Renal and urologic TB  Clinical features  Constitutional symptoms  Flank pain  Hematuria  Storage symptoms 1. Increased frequency of voiding 2. Nocturia 3. Urgency  Hypertension  Recurrent UTIs unresponsive to antibiotic therapy
  • 116. Renal and urologic TB  Diagnostics  Urine examination 1. Analysis: hematuria, sterile pyuria, proteinuria 2. Microbiology: acid-fast staining, PCR, and culture  Imaging: CT or IV urography 1. Renal scarring 2. Hydroureteronephrosis 3. Calcification involving the entire urinary tract 4. Multiple ureteral strictures 5. Bladder wall thickening and fibrosis  Cystoscopy 1. Ulceration 2. Granuloma 3. Fibrosis
  • 117. Renal and urologic TB  Treatment  Medical: See “Treatment of active TB.”  Ureteral stenting for strictures  Surgery 1. Total or partial nephrectomy 2. Bladder augmentation
  • 118. Male genital tract TB  Pathophysiology 1. Hematogenous spread of infection to the epididymis and prostate 2. Extension of infection from the epididymis to the testis, vas deferens, seminal vesicles, and ejaculatory ducts  Common sites of involvement 1. Epididymis 2. Testis 3. Prostate 4. Vas deferens, seminal vesicles, and ejaculatory duct  Clinical features 1. Scrotal mass (can be painful or painless) 2. Scrotal sinus tract with watery discharge 3. Infertility 4. Increased urinary frequency, nocturia, and hematospermia 5. Recurrent prostatitis or epididymitis unresponsive to antibiotic therapy
  • 119. Male genital tract TB  Diagnostics  Urine examination 1. Analysis: hematuria, sterile pyuria 2. Microbiology: acid-fast staining, PCR, and culture  Scrotal ultrasound 1. Calcification 2. Hydrocele 3. Diffuse or nodular enlargement of the epididymis  Transrectal ultrasound: calcification in prostate and seminal vesicles  Treatment: See "Treatment of active TB.”
  • 120. Female genital tract TB  Pathophysiology: hematogenous spread of infection as a result of primary pulmonary TB that is reactivated in immunosuppressed states  Common sites of involvement 1. Ovaries 2. Fallopian tubes 3. Endometrium  Clinical features 1. Constitutional symptoms 2. Menstrual irregularities 3. Abdominal pain 4. Infertility 5. Adnexal mass on pelvic examination
  • 121. Female genital tract TB  Diagnostics  Menstrual fluid or endometrial curettage sample: acid-fast staining, PCR, and culture  Imaging 1. Ultrasound/CT: tuboovarian abscess 2. Hysterosalpingography: occluded fallopian tubes, hydrosalpinx, calcification 3. Hysteroscopy: adhesions and obliterated uterine cavity  Treatment 1. Medical: See “Treatment of active TB.” 2. Surgery: drainage of tuboovarian abscess
  • 123. Complications 1. Rasmussen aneurysm: inflammatory pseudoaneurysm of a branch of the pulmonary artery lying adjacent to a tuberculous cavity and manifesting with hemoptysis 2. Massive hemoptysis: due to erosion of blood vessels overlying a lung cavity, Rasmussen aneurysm, or aspergilloma 3. Lung cavitation 4. Lung fibrosis 5. Aspergilloma 6. Pneumothorax 7. Bronchiectasis 8. Fibrosing mediastinitis 9. Venous thromboembolism
  • 124.
  • 125.
  • 126.
  • 127. Prevention  BCG was derived from an attenuated strain of M. bovis and was first administered to humans in 1921
  • 128. Prevention  The primary way to prevent TB is to diagnose and isolate infectious cases rapidly and to administer appropriate treatment until patients are rendered noninfectious (usually 2–4 weeks after the start of proper treatment) and the disease is cured. Additional strategies include BCG vaccination and preventive treatment of persons with TB infection who are at high risk of developing active disease.
  • 129. BCG VACCINATION 1. Historically one of the most used vaccines in the history of medicine, BCG was derived from an attenuated strain of M. bovis and was first administered to humans in 1921 2. Many BCG vaccines are available worldwide; all are derived from the original strain, but the vaccines vary in efficacy, ranging from 80% to nil. 3. These studies and a meta-analysis also found higher rates of efficacy in the protection of infants and young children from serious disseminated forms of childhood TB, such as tuberculous meningitis and miliary. 4. TB. BCG vaccine is safe and rarely causes serious complications. The local tissue response begins 2–3 weeks after vaccination, with scar formation and healing within 3 months
  • 130.
  • 131. BCG VACCINATION  Composition: live attenuated strain of M. bovis  Administration: 0.3 mL of reconstituted vaccine using a multiple puncture device over the deltoid region  Indications 1. Countries with a high TB burden: all newborns at birth 2. Countries with a low TB burden  Children with a negative TST and who have come into contact with adults with untreated/inadequately treated TB or drug-resistant TB (if the child cannot take long-term medication for infection)  Health care workers continually exposed to individuals with drug-resistant TB  Contraindications 1. Immunocompromised individuals 2. Pregnancy  Other uses: intravesical chemotherapy in superficial bladder cancer
  • 132. BCG VACCINATION  Side effects 1. most commonly, ulceration at the vaccination site and regional lymphadenitis—occur in 1–10% of vaccinated persons. 2. Some vaccine strains have caused osteomyelitis in ~1 case per million doses administered. 3. Disseminated BCG infection (“BCGitis”) and death have occurred in 1–10 cases per 10 million doses administered, although this problem is restricted almost exclusively to persons with impaired immunity, such as children with severe combined immunodeficiency syndrome or adults with HIV infection.
  • 133.
  • 134. BCG VACCINATION  BCG vaccine is recommended for routine use at birth in countries or among populations with high TB prevalence . However, because of the low risk of transmission of TB in the United States and other high-income countries, the variability in protection afforded by BCG, and its impact on the TST, the vaccine is not recommended for general use.
  • 135. Notes  BCG vaccination induces TST reactivity, which tends to wane with time.  The presence or size of TST reactions after vaccination does not predict the degree of protection afforded.
  • 137. RESEARCH AND INNOVATION  The diagnostic pipeline has expanded considerably in terms of the number of tests, products or methods in development. These include molecular tests for the detection of TB disease and drug resistance, interferon-gamma release assays (IGRAs) for the detection of TB infection, biomarker-based assays for detection of TB disease, computer-aided detection (CAD) for TB screening using digital chest radiography, and a new class of aerosol-capture technologies for detection of TB disease.  Three new antigen-based skin tests for TB infection that perform better than tuberculin skin tests (particularly in terms of specificity) were evaluated and recommended by WHO in 2022.
  • 138. RESEARCH AND INNOVATION  There were 16 vaccine candidates in clinical trials by September 2022: four in Phase I, eight in Phase II and four in Phase III. They included candidates to prevent TB infection and TB disease, and to help improve the outcomes of treatment for TB disease.  In September 2022, there were 26 drugs for the treatment of TB disease in Phase I, Phase II or Phase III trials. These drugs comprise 17 new chemical entities, two drugs that have received accelerated regulatory approval, one drug that was recently approved by the United States (US) Food and Drug Administration under the limited population pathway for antibacterial and antifungal drugs, and six repurposed drugs. There are at least 22 clinical trials to evaluate drugs and drug regimens for treatment of TB infection.
  • 139. Cases
  • 140. Case 1  A 35-year-old man comes to the physician with a 2-month history of fever, night sweats, and a cough productive of white sputum. He lives in New York and is currently homeless. He uses intravenous heroin several times a week. Three months ago he was admitted to the hospital for aspiration pneumonia. His dad passed away at the age of 54 because of “a bad heart condition.” His temperature is 38°C (100.4°F) and respirations are 22/min. Physical examination shows coarse crackles in the left upper posterior lung field. An x- ray of the chest shows a cavitary lesion with calcification in the left upper lobe. Which of the following is the most likely source of his pulmonary findings? 1. Aspiration of oral flora 2. Exposure to contaminated hot water tanks 3. Reactivation of a latent infection 4. Embolization of a bacterial vegetation
  • 141. Case 2  A 46-year-old man who is a prisoner comes to the county jail urgent care clinic because of a 1-month history of productive cough, malaise, and right-sided chest pain. During this time, he also has had shortness of breath and a 6-kg (13-Ib) weight loss. Three weeks ago, he was treated with amoxicillin for pneumonia but his symptoms did not improve. He worked as a construction worker for 15 years. He has HIV infection and hypertension. Medications include lisinopril and antiretroviral therapy. His temperature is 38.4°C (101.1°F) and blood pressure is 142/88 mm Hg. Pulse oximetry on room air shows an oxygen saturation of 91%. On pulmonary examination, scattered crackles are heard bilaterally; there is dullness to percussion at the right lung base. Chest x-ray shows right-sided hilar lymphadenopathy, a fibrous, cavitary lesion in the right lower lobe, and right-sided pleural effusion. CD4 count is 600 cells/mm3. Which of the following is the most likely diagnosis? 1. Hodking lymphoma 2. Pulmonary sarcoidosis 3. Pneumocystis Jerovecii pneumonia 4. Primary tuberculosis 5. Mesothelioma
  • 142. Case 3  A 54-year-old woman comes to the physician because of lower back pain, night sweats, and a 5-kg (11-lb) weight loss during the past 4 weeks. She has rheumatoid arthritis treated with adalimumab. Her temperature is 38°C (100.4°F). Physical examination shows tenderness over the T10 and L1 spinous processes. Passive extension of the right hip causes pain in the right lower quadrant. The patient's symptoms are most likely caused by an organism with which of the following virulence factors? 1. Polysaccharide capsule that prevents phagocytosis 2. Surface glycolipids that prevent phagolysosome fusion 3. Polypeptides that inactivate elongation factor 2 4. Proteins that bind to the Fc region of immunoglobulin G 5. Protease that cleaves immunoglobulin A
  • 143. Case 4  A previously healthy 67-year-old man comes to the physician for routine health maintenance evaluation. He works at a community center and volunteers at a local homeless shelter. A tuberculin skin test shows an induration of 14 mm. An x-ray of the chest is normal. Treatment with an antimycobacterial drug is initiated. Two months later, he has numbness and burning over both feet and an unsteady gait. Physical examination shows decreased sensation to light touch extending from the soles of the feet to the mid- shin bilaterally. This patient's condition is most likely to result in which of the following? 1. Impaired synthesis of neurotransmitters 2. Intracellular accumulation of sorbitol 3. Segmental demyelination of peripheral axons 4. Impaired breakdown of glucose to ATP 5. Degeneration of the dorsal columns
  • 144. Case 5  A 48-year-old man comes to the physician for a preplacement examination and tuberculosis test before starting a volunteer job in a refugee clinic. He has had a mild cough, sore throat, and occasional headaches for 1 week. He has hypertension and gastroesophageal reflux disease. He works as a mechanic and has never traveled outside of the US. He lives alone and has had no known contact with anyone infected with tuberculosis. He has smoked one pack of cigarettes daily for 27 years and does not drink alcohol. Current medications include ramipril and esomeprazole. He is 180 cm (5 ft 11 in) tall and weighs 89 kg (196 lb); BMI is 28 kg/m2. His temperature is 37.3°C (99.1°F), pulse is 81/min, respirations are 17/min, and blood pressure is 145/75 mm Hg. Pulmonary examination shows no abnormalities. Laboratory studies show a leukocyte count of 6200/mm3 and a hemoglobin concentration of 13.8 g/dL. A rapid test for COVID-19 is negative. A tuberculin skin test (TST) shows an induration of 13 mm in diameter after 50 hours. Which of the following is the most appropriate next step in management? 1. Chest x-ray 2. No further management is indicated 3. Interferon-y release assay 4. Repeat tuberculin skin test after 8-10 weeks 5. Isoniazid and rifapentine therapy
  • 145. Case 6  A 55-year-old woman comes to the physician because of fevers for 2 weeks. She works as a nurse and recently returned from a charity work trip to India, where she worked in a medically-underserved rural community. A tuberculin skin test 3 months ago prior to her trip showed an induration of 3 mm. Physical examination is unremarkable. An x-ray of the chest shows right-sided hilar lymphadenopathy. A sputum culture shows acid-fast bacilli. Which of the following immunologic processes most likely occurred first? 1. Transportation of bacterial peptides to regional lymph nodes 2. Formation of a nodular tubercle in the lung 3. Replication of bacteria within alveolar macrophages 4. Fusion of phagolysosomes in neutrophils 5. Production of interferon-gamma by T-helper cells 6. Migration of T-helper cells to the lungs
  • 146. Case 7  A 4-year-old boy is brought to the physician by his mother because she is worried that her child may have tuberculosis. One week ago, the patient returned from a trip to Moldova, where he visited his grandparents. The grandfather has had a cough during the past weeks and received the diagnosis of active tuberculosis (TB) 2 days ago. The child feels well. He has no history of serious illness and takes no medications. Immunizations are up- to-date. His temperature is 36.8°C (98.2°F), pulse is 88/min, respirations are 20/min, and blood pressure is 95/60 mm Hg. Physical examination and chest x-ray show no abnormalities. A tuberculin skin test (TST) shows an induration of 3 mm in diameter after 2 days. Which of the following is the most appropriate next step in management? 1. Repeat tuberculin skin test in 1 week 2. No further management is indicated at this time 3. Perform interferon-gamma release assay 4. Initiate rifampin therapy 5. Initiate rifampin, isoniazid, pyrazinamide, and ethambutol therapy